GI Medicine Flashcards

1
Q

What are the causes of Pancreatitis?

*Remember the pneumonic

What are the most common causes?

A
  • Gallstones (60%)
  • Ethanol (i.e. alcohol – 30%)
  • Trauma
  • Steroids
  • Mumps (other viruses include Coxsackie B)
  • Autoimmune (e.g. polyarteritis nodosa), Ascaris infection
  • Scorpion venom
  • Hypertriglyceridaemia, Hyperchylomicronaemia, Hypercalcaemia, Hypothermia
  • ERCP
  • Drugs (azathioprine, mesalazine*, didanosine, bendroflumethiazide, furosemide, pentamidine, steroids, sodium valproate)
    • pancreatitis is 7 times more common in patients taking mesalazine than sulfasalazine
How well did you know this?
1
Not at all
2
3
4
5
Perfectly
2
Q

Breifly explain what pancreatitis is

A

Acute pancreatitis is caused by the destructive effect of premature activation of pancreatic enzymes which causes self-perpetuating pancreatic inflammation by enzyme-mediated AUTOdigestion.

.

How well did you know this?
1
Not at all
2
3
4
5
Perfectly
3
Q

Explain pathophysiology of gall stones pancreatitis

A
  • Gallstone pancreatitis:
    • Accumulation of enzyme-rich fluid WITHIN the pancreas due to
    • OBSTRUCTION of the pancreatic duct by gallstones
    • Intracellular Ca2+ increases and causes the early activation of
    • trypsinogen
    • In this situation, trypsinogen is cleaved (by cathepsin B) to trypsin, and trypsin degradation (by chymotrypsin C) is impaired and overwhelmed leading to a buildup of trypsin and thus increased enzymatic digestion of the pancreas and inflammation leading to extensive acinar damage
How well did you know this?
1
Not at all
2
3
4
5
Perfectly
4
Q

Explain pathophysiology of gall stones pancreatitis

A
  • Alcohol-induced pancreatitis:
    • Alcohol is shown to interfere with Ca2+ homeostasis in increased stimulation of enzyme secretion and obstruction of the duct due to contraction of the ampulla of Vater
How well did you know this?
1
Not at all
2
3
4
5
Perfectly
5
Q

Explain how pancreatitis can lead to oedema, inflammation and hypovolaemia

A
  • The prematurely activate enzymes also cause leaky vessels by digesting vessel walls in the pancreas leading to the leakage of fluid into the tissues causing OEDEMA, INFLAMMATION and HYPOVOLAEMIA (as extracellular fluid is trapped in the gut, peritoneum and retroperitoneum)
  • Destruction of blood vessels by enzymes causes haemorrhage Destruction of the adjacent islets of Langerhans can result in
  • hyperglycaemia as beta cells will be destroyed resulting in less insulin
  • Lipolytic enzymes cause fat necrosis, which can result, if extensive and involving the anterior abdominal wall, in skin discolouration (Grey Turner’s sign)
    • The released fatty acids bind to Ca2+ ions, forming white precipitates in the necrotic fat
    • If this is very severe, it can result in hypocalcaemia - presenting with tetany
How well did you know this?
1
Not at all
2
3
4
5
Perfectly
6
Q

List 4 clinical signs of acute pancreatitis

A

Clinical features:

  • Severe epigastric pain that may radiate through to the back – sitting forward may relive the pain
  • Anorexia, Nausea and Vomiting - common
  • Examination may reveal guarding, tenderness, ileus and low-grade fever, tachycardia, hypotension
  • Periumbilical discolouration (Cullen’s sign) and flank discolouration (Grey-Turner’s sign) – rare
  • Other rare features:
  • ischaemic (Purtscher) retinopathy - may cause temporary or permanent blindness
How well did you know this?
1
Not at all
2
3
4
5
Perfectly
7
Q

What investigations would you request for pancreatitis and what would you expect to see? (6)

A

Investigations:

  • raised amylase:
    • 3x normal limit
    • seen in 75% of patients.
    • Note: may be normal even in severe pancreatitis as levels fall after 3-5 days of acute event & other things can cause raised amylase e.g. upper GI perforation
    • levels do not correlate with disease severity.
  • Raised serum lipase - more sensitive and specific for pancreatitis than amylase
  • Raised urinary amylase – maybe diagnostic as levels remain elevated over long time period
  • CRP level for monitoring severity and prognosis
  • ABG- for scoring and lactate
  • Erect CXR:
    • Essential to exclude gastroduodenal perforation - which also raises serum amylase
    • May show gallstones or pancreatic calcification
  • Abdominal ultrasound:
    • Diagnoses gallstone pancreatitis
  • Contrast enhanced CT:
    • To identify extent of pancreatic necrosis
  • MRI:
    • Identifies degree of pancreatic damage
    • Useful in differentiating fluid and solid inflammatory masses
How well did you know this?
1
Not at all
2
3
4
5
Perfectly
8
Q

What scoring systems are used for pancreatitis?

A

Scoring systems

  • There are several scoring systems including the Ranson score, Glasgow score and APACHE II (Acute Physiology And Chronic Health Evaluation)
  • They increase accuracy of prognosis
How well did you know this?
1
Not at all
2
3
4
5
Perfectly
9
Q

What factors indicate severe pancreaitis?

A
  • age > 55 years
  • hypocalcaemia
  • hyperglycaemia
  • hypoxia
  • neutrophilia
  • elevated LDH and AST
  • Note that the actual amylase level is not of prognostic value.
How well did you know this?
1
Not at all
2
3
4
5
Perfectly
10
Q

List 3 differential diagnoses for hyperamylasaemia aside from pancreatitis

List 3 different diagnoses for similar pain to pancreatitis

A

Differential diagnosis:

Differential causes of hyperamylasaemia

  • Acute pancreatitis
  • Pancreatic pseudocyst
  • Mesenteric infarct
  • Perforated viscus
  • Acute cholecystitis
  • Diabetic ketoacidosis

Differentials of similar pain:

  • Small bowel perforation/obstruction.
  • Ruptured or dissecting aortic aneurysm.
  • Atypical myocardial infarction.
How well did you know this?
1
Not at all
2
3
4
5
Perfectly
11
Q

What is the management of acute pancreatitis?

A

Management:

  • IV fluids:
    • Patients need prompt and adequate fluid resuscitation
    • Third space fluid losses can be huge so may require several litres stat
  • Nutrition– feed with enteral nutrition via ng tube if nutritional support is needed or person is vomiting
  • Analgesia - e.g. IM pethidine +/- iv benzodiazepine
    • Morphine is relatively contra-indicated because of possible spastic effect on the sphincter of oddi
  • Urinary catheter if unwell
  • Thromboprophylaxis - All patients with pancreatitis should have thromboprophylaxis unless clearly contraindicated (check renal function, platelets and clotting prior to dosing heparin)
  • Prophylactic antibiotic therapy – only if established infected pancreatic necrosis in the hope of averting the progression to infection.
    • Beta-lactams e.g. Iv imipenem or iv co-amoxiclav
  • Surgery
  • Patients with acute pancreatitis due to gallstones should undergo early cholecystectomy.
  • Patients with obstructed biliary system due to stones should undergo early ERCP (within 72 hours)
  • Patients who fail to settle with necrosis and have worsening organ dysfunction may require debridement, fine needle aspiration is still used by some.
  • Patients with infected necrosis should undergo either radiological drainage or surgical necrosectomy. The choice of procedure depends upon local expertise.
How well did you know this?
1
Not at all
2
3
4
5
Perfectly
12
Q

List 3 early complications of pancreatitis

A

Complications:

Early

  • Shock
  • Acute kidney injury
  • Acute respiratory distress syndrome
  • DIC
  • Sepsis
  • Hypocalcaemia
  • Hyperglycaemia
  • Pancreatic necrosis
How well did you know this?
1
Not at all
2
3
4
5
Perfectly
13
Q

List 3 late complications of pancreatitis

A

Late

  • Pancreatic necrosis
  • Pancreatic pseudocyst
  • Pancreatic fluid in lesser sac
    • Presents > 6 weeks later
    • Abdominal mass may be present
    • May need internal (via stomach) or external drainage
  • Abscess
  • Thrombosis
    • Commonly in splenic/gastroduodenal arteries
  • Fistulae
How well did you know this?
1
Not at all
2
3
4
5
Perfectly
14
Q

What are pancreatic psuedocysts?

A
  • Occur in 25% cases
  • Located in or near the pancreas and lack a wall of granulation or fibrous tissue
  • May resolve or develop into pseudocysts or abscesses
  • Since most resolve aspiration and drainage is best avoided as it may precipitate infection

Pseudocysts

  • In acute pancreatitis result from organisation of peripancreatic fluid collection. They may or may not communicate with the ductal system.
  • The collection is walled by fibrous or granulation tissue and typically occurs 4 weeks or more after an attack of acute pancreatitis
  • Most are retrogastric
  • 75% are associated with persistent mild elevation of amylase
  • Investigation is with CT, ERCP and MRI or endoscopic USS
  • Symptomatic cases may be observed for 12 weeks as up to 50% resolve
  • Treatment is either with endoscopic or surgical cystogastrostomy or aspiration
How well did you know this?
1
Not at all
2
3
4
5
Perfectly
15
Q

How would you manage pancreatic necrosis?

A

Pancreatic necrosis

  • Pancreatic necrosis may involve both the pancreatic parenchyma and surrounding fat
  • Complications are directly linked to extent of parenchymal necrosis and extent of necrosis overall
  • Early necrosectomy is associated with a high mortality rate (and should be avoided unless compelling indications for surgery exist)
  • Sterile necrosis should be managed conservatively (at least initially)
  • Some centres will perform fine-needle aspiration sampling of necrotic tissue if infection is suspected. False negatives may occur and the extent of sepsis and organ dysfunction may be a better guide to surgery
How well did you know this?
1
Not at all
2
3
4
5
Perfectly
16
Q

How would you manage a pancreatic abscess?

A

Pancreatic abscess

  • Intraabdominal collection of pus associated with pancreas but in the absence of necrosis
  • Typically occur as a result of infected pseudocyst
  • Transgastric drainage is one method of treatment, endoscopic drainage is an alternative
How well did you know this?
1
Not at all
2
3
4
5
Perfectly
17
Q

What are the causes of chronic pancreatitis?

Who are most affected and when does it typically present?

A
  • Chronic pancreatitis is an inflammatory condition which can ultimately affect both the exocrine and endocrine functions of the pancreas.
  • Around 80% of cases are due to alcohol excess with up to 20% of cases being unexplained.
  • MALES affected more than females
  • Median age of presentation is 51
  • Smoking is a risk factor

Aetiology:

  • alcohol
  • genetic: cystic fibrosis, haemochromatosis
  • ductal obstruction: tumours, stones, structural abnormalities including pancreas divisum and annular pancreas
  • recurrent acute pancreatitis
How well did you know this?
1
Not at all
2
3
4
5
Perfectly
18
Q

What is the pathophsyiology of pancreatitis?

A

Pathophysiology:

  • Obstruction or reduction in bicarbonate secretion, which produces an alkaline pH which in turn stabilises trypsinogen, leads to the activation of trypsinogen as pH rises making it more unstable and causing its activation into trypsin which leads to pancreatic tissue necrosis with eventual fibrosis
  • Abnormalities of bicarbonate excretion can be the result of functional defects at the level of the cellular wall e.g. cystic fibrosis or mechanical as in trauma
  • Increased intrapancreatic enzyme activity leads to the precipitation of proteins within the duct lumen in the form of plugs
  • These plugs then become calcified resulting in ductal obstruction and further pancreatic damage
  • Alcohol increases trypsinogen activation and also causes proteins to precipitate in the ductal structure of the pancreas leading to local pancreatic dilatation and fibrosis
  • NOTE: the vast majority of people drinking excess alcohol DO NOT DEVELOP pancreatitis - this suggests that the disease process is a complex interaction of different mechanisms e.g. genes and environment
How well did you know this?
1
Not at all
2
3
4
5
Perfectly
19
Q

List 5 clinical features of chronic pancreatitis?

A

Clinical Features

  • epigastric pain is typically worse 15 to 30 minutes following a meal
    • exacerbated by alcohol, made better by sitting forward
    • pain ‘bores’ to the back
  • Nausea and vomiting
  • Anorexia
  • Diarrhoea
  • steatorrhoea: symptoms of pancreatic insufficiency usually develop between 5 and 25 years after the onset of pain (exocrine dysfunction)
  • weight loss
  • diabetes mellitus develops in the majority of patients. It typically occurs more than 20 years after symptom begin (endocrine dysfunction)
How well did you know this?
1
Not at all
2
3
4
5
Perfectly
20
Q

List 4 differential diagnoses of chronic pancreatitis

A

Differentials:

  • Acute pancreatitis
  • Pancreatic malignancy
  • Acute cholecystitis.
  • Peptic ulcer disease.
  • Acute hepatitis.
  • Abdominal aortic aneurysm.
  • Pyelonephritis.
How well did you know this?
1
Not at all
2
3
4
5
Perfectly
21
Q

What investigations would you request for chronic pancreatitis and what would you expect to see?

A

Investigations:

  • Amylase and lipase usually normal as there may not be sufficient residual acinar cells to produce elevation
  • Fasting blood glucose – check for DM
  • abdominal x-ray shows pancreatic calcification and dilated pancreatic duct
  • CT is more sensitive at detecting pancreatic calcification.
  • MRCP – detects more subtle abnormalities
  • EUS – endoscopic ultrasound can allow for direct visualisation of the pancreas
  • faecal elastase may be used to assess exocrine function if imaging inconclusive
  • Secretin stimulation test: a positive if 60%+ pancreatic exocrine function is damaged
  • Biopsy rarely performed – too risky
How well did you know this?
1
Not at all
2
3
4
5
Perfectly
22
Q

What is the management of chronic pancreatitis?

A

Management

  • Stop alcohol, stop smoking and high protein low carb diet
  • pancreatic enzyme supplements e.g. Creon
  • analgesia for Abdo. pain e.g. NSAIDS or paracetamol. Or coeliac plexus block
  • Insulin – if diabetic
  • Pancreatic duct stones - dislodge with ERCP or may require endoscopic shock-wave lithotripsy or laser lithotripsy.
  • Surgical decompression of duct dilatation can be performed if this cannot be achieved by ERCP alone
  • Local surgical resection of pancreatoduodenectomy has been used in some cases
How well did you know this?
1
Not at all
2
3
4
5
Perfectly
23
Q

List 3 complications of chronic pancreatitis

A

Complications:

  • Malabsorption
  • Diabetes
  • Chronic pain
  • Pancreatic pseudocyst
    • These can rupture, bleed, or occlude nearby structures like the duodenum or CBD. If present for >6 weeks, spontaneous resolution is unlikely and they should be drained, either surgically or endoscopically into the stomach or duodenum.
  • Ascites or pleural effusions if pancreatic duct is occluded
    • Ascitic or pleural amylase will be elevated
  • Pancreatic carcinoma
How well did you know this?
1
Not at all
2
3
4
5
Perfectly
24
Q

What is the prognosis of patients with chronic pancreatitis?

A

Prognosis:

  • There is an increased mortality and morbidity
  • Approximately 1/3 of patients will die within 10 years
How well did you know this?
1
Not at all
2
3
4
5
Perfectly
25
Q

What is autoimmune pancreatitis?

How does it present?

Who does it mostly affect?

What investigations would you request and how is it treated?

A

AUTOIMMUNE CHRONIC PANCREATITIS:

  • Chronic pancreatic inflammation which results from an autoimmune process
  • High prevalence in Japan
  • Presentation is very similar to normal chronic pancreatitis
  • There are elevated levels of serum gammaglobulins and immunoglobulin G (IgG) levels - IgG4
  • Autoantibodies such as antinuclear cytoplasmic antibody (ANCA) and rheumatoid factor may also be elevated
  • This condition is steroid responsive with glucocorticoid therapy e.g. ORAL PREDNISOLONE for 4-6 weeks
How well did you know this?
1
Not at all
2
3
4
5
Perfectly
26
Q

What is the function of the liver? (4)

A

Functions:

  • Glucose & fat metabolism
  • Detoxification and excretion:
  • Bilirubin
  • Ammonia
  • Drugs/hormones/pollutants
  • Protein synthesis:
    • Albumin
  • Clotting factors
  • Defence against infection (Reticulo-endothelial system)
How well did you know this?
1
Not at all
2
3
4
5
Perfectly
27
Q

List 3 causes of acute liver damage

List 3 causes of chronic liver damage

A
How well did you know this?
1
Not at all
2
3
4
5
Perfectly
28
Q

List 5 signs of acute liver damage

A
  • Acute:
  • Malaise, nausea, anorexia
  • Occasionally jaundice (doesn’t occur with everyone)
  • Rare:
    • Confusion (encephalopathy)
    • Bleeding
    • Liver pain
    • Hypoglycaemia (since liver breaks down glycogen to glucose, also the liver is the bodies major glucose store)
How well did you know this?
1
Not at all
2
3
4
5
Perfectly
29
Q

List 5 signs of chronic liver damage

A
  • Chronic:
  • Ascites (fluid accumulation in the peritoneal cavity)
  • Oedema, Haematemesis (varices), malaise, anorexia, wasting, easy bruising (since liver produces clotting factors), itching, hepatomegaly, abnormal LFTs
  • Rare:
    • Jaundice
    • Confusion
How well did you know this?
1
Not at all
2
3
4
5
Perfectly
30
Q

What blood tests are used to assess liver damage?

A

Key Points:

  • ALT + AST elevation indicates hepatocellular damage
  • An AST:ALT ratio greater than 2:1 indicates alcoholic aetiology of hepatocellular damage – typically ALT>AST in non-alcoholic liver disease
  • ALP + GGT elevation indicates cholestasis
  • ALP elevation alone may not be liver disease – it can also be elevated due to osteoblast activity (i.e. children and adolescents growing rapidly, people with fractures, perimenopausal women, people with Paget’s disease of the bone), during the third trimester of pregnancy and temporarily following ingestion of fatty foods in people with blood type O
  • GGT will often also rise acutely when there is significant consumption of alcohol
  • Albumin, bilirubin and prothrombin time give an indication of synthetic function in the liver. If you know there is damage to the liver, they are helpful at evaluating the degree of damage
  • Serum albumin:
  • Marker of synthetic function and is useful for gauging the severity of chronic liver disease: a falling serum albumin is a bad prognostic sign
  • In acute liver disease, initial albumin levels may be normal
  • Prothrombin time:
  • Marker of synthetic function
  • Due to its short half-life it is a sensitive indicator of both acute and chronic liver disease
  • Vitamin K deficiency can cause a prolonged prothrombin time and commonly occurs in biliary obstruction, as the low concentration of bile salts result in poor absorption of vitamin K
  • Serum bilirubin:
    • Marker for liver excretory function
  • Liver biochemistry - GIVE NO INDEX of LIVER FUNCTION:
    • Aminotransferases:
      • These enzymes are contained in hepatocytes and leak into the blood with liver cell damage
      • ALT>AST = Hepatitis
      • AST> ALT = alcohol or advanced cirrhosis/fibrosis
    • Aspartate aminotransferase (AST):
    • Also present in heart, muscle, kidney and brain
    • High levels are seen in hepatic necrosis, myocardial infarction, muscle injury and congestive cardiac failure
    • Alanine aminotransferase (ALT):
    • More specific to the liver
    • Rise can occur in thyroid disease, coeliac disease or polymyositis
  • Alkaline phosphate:
  • Present in liver but also in bone, intestine and placenta
  • Raised in both intrahepatic and extrahepatic cholestatic disease of any cause, due to increased synthesis
  • Raised levels also occur with hepatic infiltrations (e.g. metastases) and in cirrhosis
  • High ALP but normal GGT = bones, pregnancy, vit D deficiency
  • High ALP and High GGT = biliary source
How well did you know this?
1
Not at all
2
3
4
5
Perfectly
31
Q

Outline what is involved in NILS

A

Non- invasive liver screen (NILS):

  • Liver USS
  • Bloods:
    • liver function tests - including gamma GT and total protein
    • coagulation tests, including INR and APTT
    • hepatitis serology - for B (HBsAg) and C (HCV Ab)
    • ferritin
    • transferrin saturation
    • alpha 1 antitrypsin
    • alpha-feto protein
    • serum copper, ceruloplasmin
    • immunoglobulins:
      • IgG - autoimmune hepatitis
      • IgM – primary biliary cirrhosis
      • IgA - alcohol
      • All 3 – cirrhosis
    • autoantibody screen:
      • ANA/ASMA – autoimmune hepatitis (type1)
      • LKM - autoimmune hepatitis (type 2)
      • AMA (M2) – Primary biliary cirrhosis
      • ANCA – primary sclerosing cholangitis
How well did you know this?
1
Not at all
2
3
4
5
Perfectly
32
Q

Explain how bilirubin and haem are metabolised

A

Bilirubin & haem catabolism:

  • Bile pigments are substance formed from the haem portion of haemoglobin when old/damaged erythrocytes are broken down in the spleen and liver
  • The predominant bile pigment is bilirubin, which is extracted from the blood by the hepatocytes and actively secreted into bile
  • Bilirubin is yellow and contributes to the colour of bile
  • Old/damaged erythrocytes are broken down by macrophages in the spleen and bone marrow but also in the kupffer cells (resident macrophages) of the liver
  • When the erythrocytes is ingested it is broken down into haem and globin
  • Globin (a protein) is broken down into amino acids which can then be used to generate new erythrocytes in the bone marrow
  • Haem is further broken down into biliverdin , Fe2+ (transported to bone marrow to be implemented into new erythrocytes by transporter transferrin) and CO
  • Biliverdin is reduced by biliverdin reductase into UNCONJUGATED BILIRUBIN - this is toxic and must be secreted, it is lipid soluble and thus insoluble in blood and must be transported bound to albumin to the liver
  • In the liver it undergoes glucuronidation - the addition of a glucuronic acid in order to make it soluble to be excreted under the action of UDP Glucuronyl Transferase (in Gilbert’s this enzyme is deficient resulting in raised unconjugated bilirubin) which converts it to CONJUGATED BILIRUBIN
  • The conjugated bilirubin travels to the small intestine until it reaches the ileum or the beginning of the large intestine where under the action of intestinal bacteria it is reduced through a hydrolysis reaction (a glucuronic acid group is removed) forming urobilinogen
  • Urobilinogen is lipid soluble, around 10% is reabsorbed into the blood and bound to albumin and transported back to the liver - urobilinogen oxidised to urobilin
  • Here it is either re-cycled into bile or transported into the kidneys where it is excreted in urine- responsible for the yellowish colour of urine
  • The remaining 90% of urobilinogen is oxidised by a different type of intestinal bacteria to form stercobilin
  • Stercobilin is then excreted into the faeces - responsible for its brownish colour
How well did you know this?
1
Not at all
2
3
4
5
Perfectly
33
Q

List 5 causes of pre-hepatic jaundice

A

Pre-hepatic causes of jaundice

  • Congenital red cell issues
    • Cell shape
      • Sickle cell disease
      • Hereditary spherocytosis
      • Hereditary elliptocytosis
    • Enzyme
      • G6PD deficiency
      • Pyruvate kinase deficiency
    • Haemoglobin
      • Thalassemia
  • Autoimmune haemolytic anaemia
  • Drugs
    • Penicillins
    • Sulphasalazine
  • Infections
    • Malaria
  • Mechanical
    • Metallic valve prostheses
    • DIC
  • Transfusion reactions
  • Paroxysmal nocturnal haemoglobinuria
How well did you know this?
1
Not at all
2
3
4
5
Perfectly
34
Q

List 5 causes of hepatic jaundice

A

Hepatic causes of jaundice

  • Conjugated causes
    • Cirrhosis (see chronic liver disease for further causes)
    • Malignancy
      • Primary or metastases
    • Viral hepatitis
    • Drugs
      • Hepatitis
        • Isoniazid, rifampicin, atenolol, enalapril, verapamil, nifedipine, amiodarone, ketoconazole, cytotoxics, halothane
      • Cholestasis
        • Ciclosporin, azathioprine, chlorpromazine, cimetidine, erythromycin, nitro, ibuprofen, hypoglycaemics
    • Enzymes
      • Dubin-Johnson syndrome (DJS)
        • Autosomal recessive (cMOAT gene) with excretion of conjugated bilirubin.
          • Leads to pigmented liver.
        • Increase in conjugated bilirubin with no other enzyme changes
        • High coproporphyrin
      • Rotor syndrome
        • Similar to DJS
        • Liver not pigmented
        • Normal coproporphyrin
How well did you know this?
1
Not at all
2
3
4
5
Perfectly
35
Q

List 5 causes of Unconjugated jaundice ?

A
  • Unconjugated causes of jaundice (sometimes classified as pre-hepatic causes)
    • Gilbert’s syndrome
      • Congenital hypo-activity of conjugation enzyme UGT-1. Benign and common (5%)
      • Normal LFTs except mildly elevated bilirubin, especially in times of physiological stress/illness
      • Normal life expectancy
    • Crigler-Najar syndrome
      • Autosomal recessive (type I) or dominant (type II). Severe unconjugated hyperbilirubinaemia.
      • Congenital absence (I) or decrease (II) of glucoronyl transferase.
      • Normal liver histology.
      • Treatment is liver transplant (only type II survive to adulthood)
How well did you know this?
1
Not at all
2
3
4
5
Perfectly
36
Q

List 5 causes of Post-hepatic causes of jaundice

A

Post-hepatic causes of jaundice

  • Biliary tree obstruction
    • Gallstones
    • Compression e.g. pancreatitis, pancreatic tumour, lymph nodes, biliary atresia
    • Cholangiocarcinoma
    • Post-operative stricture
  • Primary biliary cirrhosis
    • M:F = 1:9
    • ANA and Anti- mitochondrial antibodies
      • And anti-centromere for prognosis (though more association with CREST)
  • Primary sclerosing cholangitis
    • 80% of PSC have UC
    • ANCA, anti-smooth muscle antibodies

Association with cholangiocarcinoma

How well did you know this?
1
Not at all
2
3
4
5
Perfectly
37
Q

List 5 causes of Pregnancy-associated jaundice

A

Pregnancy-associated jaundice

  • Obstetric cholestasis
    • 0.1-0.2% of pregnancies
    • Presentation
      • Itching – jaundice later
      • Raised liver markers, esp ALP
    • Issues
      • Fetal mortality 3.5%
    • Often recurs in further pregnancies
    • Treatment
      • Ursodeoxycholic acid
  • HELLP (Haemolysis, Elevated Liver enzymes, Low Platelets)
    • Occurs in 1-2 out of 1000 pregnancies and 10-20% of severe pre-eclampsia
    • Leads to a variant of DIC
    • Needs steroids and prompt delivery
    • Maternal mortality 1-24%
  • Fatty liver of pregnancy
    • All LFTs including synthetic function go off
  • Hyperemesis gravidum
  • Pre-eclampsia
    • Associated with abnormal LFTs in 20% cases
How well did you know this?
1
Not at all
2
3
4
5
Perfectly
38
Q

Describe how post hepatic or hepatic jaundice may present

A

Dark urine

Pale stools if post hepatic

Itching

Abnormal LFTs

How well did you know this?
1
Not at all
2
3
4
5
Perfectly
39
Q

How would bilirubin, ALT and AST levels present in :

Pre-hepatic jaundice

Hepatic jaundice

Post-hepatic jaundice

A
How well did you know this?
1
Not at all
2
3
4
5
Perfectly
40
Q

What is biliary colic?

A

Biliary colic

  • The term used for the pain associated with the temporary obstruction of the cystic or common bile duct by a stone migrating from the gall bladder
How well did you know this?
1
Not at all
2
3
4
5
Perfectly
41
Q

What are the risk factors biliary colic?

A

Risk factors

  • it is traditional to refer to the ‘4 F’s’:
    • Fat: obesity is thought to be a risk factor due to enhanced cholesterol synthesis and secretion
    • Female: gallstones are 2-3 times more common in women. Oestrogen increases activity of HMG-CoA reductase
    • Fertile: pregnancy is a risk factor
    • Forty
  • other notable risk factors include:
    • diabetes mellitus
    • Crohn’s disease
    • rapid weight loss e.g., weight reduction surgery
    • drugs: fibrates, combined oral contraceptive pill
How well did you know this?
1
Not at all
2
3
4
5
Perfectly
42
Q

Explain the pathophysiology of biliary colic

A

Pathophysiology

  • occur due to ↑ cholesterol, ↓ bile salts and biliary stasis
  • the pain occurs due to the gallbladder contracting against a stone lodged in the cystic duct
How well did you know this?
1
Not at all
2
3
4
5
Perfectly
43
Q

Describe the features of biliary colic

A

Features

  • colicky right upper quadrant abdominal pain
    • sudden onset – crescendo character
    • worse postprandially, worse after fatty foods
    • the pain may radiate to the right shoulder/interscapular region
  • nausea and vomiting are common
  • Around 15% of patients are found to have gallstones in the common bile duct (choledocholithiasis) at the time of cholecystectomy, This can result in obstructive jaundice in some patients
How well did you know this?
1
Not at all
2
3
4
5
Perfectly
44
Q

What complications may arise from biliary colic?

A

Possible complications other than biliary colic

  • acute cholecystitis: the most common complication
  • ascending cholangitis
  • acute pancreatitis
  • gallstone ileus
  • gallbladder cancer
How well did you know this?
1
Not at all
2
3
4
5
Perfectly
45
Q

What is acute choleystitis?

A

ACUTE CHOLECYSTITIS

  • Acute cholecystitis describes inflammation of the gallbladder.
  • develops secondary to gallstones in 90% of patients (acute calculous cholecystitis)- blockage of the cystic duct or neck of gallbladder so there is obstruction of gallbladder emptying
  • the remaining 10% of cases are referred to as acalculous cholecystitis
    • typically seen in hospitalised and severely ill patients
    • multifactorial pathophysiology: gallbladder stasis, hypoperfusion, infection in immunosuppressed patients it may develop secondary to Cryptosporidium or cytomegalovirus
    • RF: sepsis, burns, trauma, AKI, DM, TPN, starvation
    • associated with high morbidity and mortality rates
How well did you know this?
1
Not at all
2
3
4
5
Perfectly
46
Q

Describe the pathophsyiology of cholecystitis?

A

Pathophysiology:

  • Two types of gallstone; cholesterol gallstone and bile pigment stones

Cholesterol gallstone:

  • Accounts for the majority (80%) of gallstones in the Western world
  • Large stones that are often solitary
  • Main causes are being female, age and obesity
  • Cholesterol stone formation due to cholesterol crystallisation in bile
  • Cholesterol is held in solution by the detergent action of bile salts and phospholipids, with which it forms micelles and vesicles
  • Cholesterol gallstones only form in bile which has an EXCESS of cholesterol, either because there is a:
  • Relative deficiency in bile salts and phospholipids
  • Relative excess of cholesterol (supersaturated or lithogenic bile) e.g. in diabetes mellitus or in a high cholesterol diet (also decreases bile salt synthesis)
  • NOTE: many people with supersaturated bile may never develop stones - it is the balance between cholesterol crystallising and solubilising factors that determines whether cholesterol will crystallised out of solution
  • Other factors that determine gallstone formation:
  • Reduced gallbladder motility and stasis e.g. in pregnancy and
  • diabetes
  • Crystalline promoting factors in bile e.g. mucus and calcium
  • Most of cholesterol is derived from hepatic uptake from diet:
  • Hepatic biosynthesis of cholesterol only accounts for 20%

Bile pigment stones - mainly formed of Ca2+:

  • Pathogenesis is completely independent of cholesterol gall stones • Small stones that are friable and irregular
  • Main cause is haemolysis
  • Two main types; black and brown
  • Black pigment gallstones:
    • Calcium bilirubinate composition and a network of mucin glycoproteins that interlace with salts e.g. calcium bicarbonate
    • Glass-like cross-sectional surface
    • Seen a lot in patients with haemolytic anaemias e.g. spherocytosis, sickle cell and thalassaemia - chronic excess of bilirubin
  • Brown pigment stones:
  • Composed of calcium salts e.g. calcium bicarbonate, fatty acids and calcium bilirubinate
  • Muddy hue with an alternating brown and tan layer on cross-section
  • Almost always found in the presence of bile stasis and/or biliary infection
  • Common cause of recurrent bile duct stones following cholecystectomy
How well did you know this?
1
Not at all
2
3
4
5
Perfectly
47
Q

List 5 clinical features of cholecystitis

A

Clinical features

  • Right upper quadrant pain
  • May radiate to the right shoulder (due to irritation of diaphragm)
  • Fever and signs of systemic upset
  • Murphy’s sign on examination: inspiratory arrest upon palpation of the right upper quadrant
  • Liver function tests are typically normal
  • Deranged LFTs may indicate Mirizzi syndrome - a gallstone impacted in the distal cystic duct causing extrinsic compression of the common bile duct
  • Jaundice (in <10 % of patients)
  • Leucocytosis – in cholecystitis but not in biliary colic
How well did you know this?
1
Not at all
2
3
4
5
Perfectly
48
Q

What investigations would you request for cholecystitis and what would you expect to see?

A

Investigations:

  • Bloods:
    • Full blood count – ↑ESR, ↑CRP, ↑WCC
    • Serum bilirubin, alkaline phosphatase and amino transferase may all be slightly raised.
    • Serum amylase – ↑- acute pancreatitis may be present as a complication of gallstones.
  • Abdominal ultrasound is the first-line investigation of choice
  • Will detect Gallstones, Gallbladder wall thickening, Dilated common bile duct (>6mm)
  • If still unclear use cholescintigraphy (HIDA scan) may be used
    • technetium-labelled HIDA (hepatobiliary iminodiacetic acid) is injected IV and taken up selectively by hepatocytes and excreted into bile
    • in acute cholecystitis there is cystic duct obstruction (secondary to oedema associated with inflammation or an obstructing stone) and hence the gallbladder will not be visualised
How well did you know this?
1
Not at all
2
3
4
5
Perfectly
49
Q

What is the management of cholecystitis?

A

Management:

  • Nil By Mouth
  • Bed rest
  • Antibiotics:
    • Co-amoxiclav– IV
    • Metronidazole – add if patient is particularly ill
      • Bacteria associated with cholecystitis are:
      • Klebsiella, Enterococcus and Escheria coli (E.coli)
  • Analgesics: usually diclofenac (NSAID), with pethidine (fast acting opioid) in more severe cases.
  • IV fluids
  • Once stable - early laparoscopic cholecystectomy, within 1 week of diagnosis.
  • Percutaneous cholecystostomy – may be suitable for some (e.g. old/frail), and still allows for future cholecystectomy.
How well did you know this?
1
Not at all
2
3
4
5
Perfectly
50
Q

List 4 complications cholecystitis

A

Complications:

  • Necrosis of the gallbladder wall (gangrenous cholecystitis).
  • Perforation of the gallbladder.
  • Biliary peritonitis.
  • Peri-cholecystic abscess.
  • Fistula (between the gallbladder and duodenum).
  • Jaundice (due to inflammation of adjoining biliary ducts — Mirizzi’s syndrome).
  • Bacterial infection and subsequent empyema
    • Bacterial Infection – is a consequence and not a cause of cholecystitis.
    • Empyema – this is a collection of pus in a bodily cavity. It is different from an abscess, which is a collection of pus in a newly formed body cavity
How well did you know this?
1
Not at all
2
3
4
5
Perfectly
51
Q

What is ascending cholangitis?

A

ASCENDING CHOLANGITIS

  • Ascending cholangitis is a inflammation or bacterial infection (typically E. coli) of the biliary tree. The most cause is Obstruction of biliary tree secondary to gallstones
  • Other common causes:
    • Benign biliary strictures following biliary surgery
    • Infection post ERCP
    • Invasion by tumour
      • Pancreatic, cholangiocarcinoma, hepatocellular carcinoma, metastases
    • Roundworm or liver fluke infection (common overseas) – which cause blockage
    • HIV cholangiopathy
  • It is rare but can be 1-2% post ERCP
How well did you know this?
1
Not at all
2
3
4
5
Perfectly
52
Q

What are the clinical features of ascending cholangitis?

A

Clinical features:

  • Charcot’s triad of right upper quadrant (RUQ) pain, fever and jaundice occurs in about 20-50% of patients
    • fever is the most common feature, seen in 90% of patients- with rigors
    • RUQ pain 70%
    • jaundice 60% - it is cholestatic jaundice so there dark urine, pale stools
  • hypotension and confusion (Due to sepsis) are also common (the additional 2 factors in addition to Charcot’s triad make Reynolds’ pentad)
  • Peritonism is uncommon and suggests alternative cause, e.g. appendix or ruptured gall bladder
How well did you know this?
1
Not at all
2
3
4
5
Perfectly
53
Q

List 3 DD of ascending cholangitis

A

Differential:

  • Cholecystitis
  • Other causes of acute jaundice
  • CBD gallstone causing obstructive jaundice
How well did you know this?
1
Not at all
2
3
4
5
Perfectly
54
Q

What investigations would you request for ascending cholangitis and what would you expect to see?

A

Investigations:

  • Blood tests:
    • Full blood count -HIGH neutrophil count
    • Urea and electrolytes
    • Clotting
    • Amylase
    • Inflammatory markers – RAISED
    • Serum bilirubin – raised due to bile duct obstruction
    • Aminotransferases – raised (ALT higher than AST)
    • Alkaline phosphate – raised
  • Blood cultures
    • Usually gram-negative: E.coli, Klebsiella, Enterobacter
  • Imaging:
    • AXR – may show ileus or air in biliary tree (e.g. after ERCP; gas-producing organisms; cholecystenteric-fistula)
    • USS – gallstones or dilated ducts – initial imaging of choice
    • MRCP (Magnetic resonance cholangiopancreatography):
      • Clearly shows biliary tree making detection of common bile duct stones and presence or dilated duct much more clearer to see
    • CT abdomen – excludes pancreatic cancer and easier to spot pigmented stones
How well did you know this?
1
Not at all
2
3
4
5
Perfectly
55
Q

What is the management of of ascending cholangitis?

A

Management:

  • intravenous antibiotics CEFOTAXIME and METRONIDAZOLE - continued after biliary drainage until symptom resolution
  • IV fluid and catheterization for fluid balance
  • biliary drainage via endoscopic retrograde cholangiopancreatography (ERCP) after 24-48 hours to relieve any obstruction
    • ERCP can be diagnostic and therapeutic as stones can be removed and a sphincterotomy performed at the Sphincter of Oddi to prevent future episode
  • Cholecystectomy
    • All patients with an episode of biliary sepsis secondary to gallstones should be referred to the surgeons for consideration of an elective cholecystectomy once recovered.
56
Q

What are the complications of ascending cholangitis?

A

Complications:

  • Septic shock and death
  • Intra-abdominal collection
57
Q

Describe the prognosis of patients with ascending cholangitis

A

Prognosis:

  • Acute cholangitis has a high mortality (7-40%),
  • Higher mortality in patients with co-morbidities, e.g. elderly, renal failure, cirrhosis, metastatic disease, failure to respond to antibiotics
58
Q

How do you differentiate between colic, cholecystitis and cholangitis?

A
59
Q

What is the difference between cirrhosis and chronic liver disease?

A
  • Cirrhosis is not a specific disease; it is an end stage of all progressive chronic liver diseases; which once fully developed is irreversible and may be associated clinically with symptoms and signs of liver failure and portal hypertension
  • It is irreversible liver damage
  • Histologically, there is loss of normal hepatic architecture with bridging fibrosis and nodular regeneration
  • Chronic liver disease (CLD)is progressive destruction of the liver parenchyma over a period greater than 6 months leading to fibrosis and cirrhosis
60
Q

What are the causes of chronic liver disease and cirrhosis?

A

Aetiology:

Common:

  • Alcohol abuse (most common)
  • non-alcoholic fatty liver disease (NAFLD)
  • viral hepatitis (B and C)

Others:

  • Primary biliary cirrhosis
  • Primary sclerosing cholangitis
  • Autoimmune hepatitis - presents as high ALT
  • Hereditary haemochromatosis (iron overload)
  • Wilson’s disease
  • Alpha-Antitrypsin deficiency
  • Drugs e.g. amiodarone, methotrexate, phenytoin, sodium valproate, isoniazid, nitrofurantoin
  • Budd- Chiari
61
Q

What are the features of CLD?

A

Features of CLD:

  • Fatigue
  • Encephalopathy
  • Oedema
  • Bruising
  • Hands
    • Dupuytren’s contracture, palmar erythema, leukonychia (white discolouration on nails due to hypoalbuminemia), clubbing
  • Asterixis
  • Face
    • Jaundiced sclera
    • Fetor hepaticus
    • Xanthelasma - yellow fat deposits under skin usually around eyelids
  • Chest
    • Spider naevi, gynaecomastia
  • Abdomen
    • Hepatomegaly, splenomegaly (due to portal hypertension)
    • Ascites, caput medusa
  • Polyneuropathy
62
Q

Explain the pathophysiology of CLD and name the two types of cirrhosis

A

Pathophysiology:

  • Chronic liver injury results in inflammation, matrix deposition, necrosis and angiogenesis all of which lead to FIBROSIS
  • Liver injury causes necrosis and apoptosis, releasing cell contents and reactive oxygen species (ROS)
  • This activates hepatic stellate cells and tissue macrophages (Kupffer cells)
  • Stellate cells release cytokines that attract neutrophils and macrophages to the liver which results in further inflammation and thus necrosis and eventual fibrosis
  • Kupffer cells phagocytose necrotic and apoptotic cells and secrete pro- inflammatory mediators:
    • Transforming growth factor-beta (TGF-beta) which leads to the transdifferentiation of stellate cells to myofibroblasts
    • Platelet derived growth factor (PDGF) which stimulates myofibroblast proliferation
    • Increased myofibroblasts leads to progressive collagen matrix deposition resulting in fibrosis and scar accumulation in the liver
  • This results in severe reduction in liver function as fibrosis is non- functioning
  • If the cause of fibrosis is eliminated e.g. treatment of viral hepatitis then resolution (complete reversal to near-normal architecture) of early fibrosis can occur
  • In cirrhosis, regression (improvement, not reversal) occurs, which improves clinical outcomes
  • The characteristic features of cirrhosis are regenerating nodules separated by fibrous septa and loss of lobular architecture within the nodules
  • Two types:
    • Micronodular cirrhosis:
      • Regenerating nodules are usually < 3mm in size with uniform involvement of the liver
      • Often caused by alcohol or biliary tract disease
    • Macronodular cirrhosis:
      • There nodules are of varying size and normal acini (functioning unit of liver) may be seen within the larger nodules
      • Often caused by chronic viral hepatitis
63
Q

What investigations would you request for CLD and expect to see?

A

Investigations:

Bloods:

  • FBC: normocytic normochromic anaemia with leukopenia and thrombocytopenia often present
  • LFTs: deranged across the board. Note can be normal in very advanced disease.
  • Coagulation screen
  • Note that the tests of synthetic liver function are albumin, PT and platelets
    • If these are deranged suspect severe disease
  • U+Es can be deranged in hepatorenal syndrome or excess diuretic therapy in response to ascites and oedema. Low NA+ indicates severe liver disease
  • Viral screen: Hep B,C and delta, HIV 1&2
  • Autoantibodies (PBC/PSC)
  • Serum Immunoglobulins
  • Iron studies and ferritin (Haemochromatosis)
  • Copper and Ceruloplasmin (Wilson’s)
  • Alpha-1 antitrypsin level (deficiency)
  • Imaging: CT and USS – these will show Can demonstrate fatty liver, nodularity of cirrhosis, distortion of architecture, Hepatosplenomegaly and can detect HCC
  • traditionally a liver biopsy was used -this procedure is however associated with adverse effects such as bleeding and pain
  • transient elastography and acoustic radiation force impulse imaging are increasingly used – NICE recommended to diagnose cirrhosis
  • for patients with NAFLD, NICE recommend using the enhanced liver fibrosis score to screen for patients who need further testing
  • Further investigations:
    • upper endoscopy to check for varices in patient’s with a new diagnosis of cirrhosis
      • If varices present patient should be enrolled on a banding programme or commenced on a non-selective beta-blocker (e.g. propranolol)
    • liver ultrasound every 6 months (+/- alpha-feto protein) to check for hepatocellular cancer

transient elastography:

  • brand name ‘Fibroscan’
  • uses a 50-MHz wave is passed into the liver from a small transducer on the end of an ultrasound probe
  • measures the ‘stiffness’ of the liver which is a proxy for fibrosis
64
Q

Describe what is involved in liver cirrhosis screening

A

Cirrhosis screening: NICE made a specific recommendation, suggesting to offer transient elastography to:

  • people with hepatitis C virus infection
  • men who drink over 50 units of alcohol per week and women who drink over 35 units of alcohol per week and have done so for several months
  • people diagnosed with alcohol-related liver disease
65
Q

What scoring system is used for cirrhosis?

A

Scoring system:

  • For many years the Child-Pugh classification was used to classify the severity of liver cirrhosis.
  • In recent years the Model for End-Stage Liver Disease (MELD) has been increasingly used, particularly patient’s who are on a liver transplant waiting list
  • There is also another scoring system called UKELD (UK model of end-stage liver disease) which uses:
  • Bilirubin
  • Creatinine
  • INR
  • Sodium
66
Q

What is the management of CLD and cirrhosis?

A

Management:

  • Abstinence from alcohol
    • Important for other causes of cirrhosis, not just alcohol-related CLD
  • Treat the underlying cause of CLD e.g. Antiviral treatment; steroids; ursodeoxycholic acid etc
  • 6 monthly USS (3-monthly if haemochromatosis) and AFP to screen for hepatocellular carcinoma
  • Avoid NSAIDs and aspirin as these may precipitate gastro-intestinal bleeding or renal impairment
  • If very advanced and no longer responsive to therapy then liver transplantation
  • Treat complications and decompensations
67
Q

What are the complications of CLD and cirrhosis?

A

Complications:

  • Coagulopathy; fall in clotting factors II,VII, IX & X
  • Hepatic Encephalopathy - liver flap (asterixis - flapping tremor with wrist extended) & confusion/coma
  • hypoalbuminemia resulting in oedema
  • Portal hypertension:
    • Ascites – may lead to spontaneous bacterial peritonitis
    • Oesophageal varices
  • HCC
  • Hepatorenal syndrome

Prognosis: Poor prognostic factors include grade III or IV encephalopathy, age >40, drug-induced hepatic failure, high INR

68
Q

How is alcohol metabolised?

A

Physiology of alcohol metabolism:

  • Ethanol is metabolised in the liver by two pathways, resulting in the increase in the NADH/NAD ratio
  • The altered redox potential causes increased hepatic fatty aid synthesis with decreased fatty acid oxidation - this results in the hepatic accumulation of fatty acids which are then esterified to glycerides
  • The changes in oxidation-reduction also impair carbohydrate and protein metabolism and are the cause of the centrilobular necrosis of the hepatic acinus that is typical of alcohol damage
  • Tumour necrosis alpha (TNF-alpha) release from Kupffer cells causes the release of reactive oxygen species (ROS), leading to tissue injury and fibrosis
  • Acetaldehyde is formed by the oxidation of ethanol, and its effect on hepatic proteins could be a factor in producing liver cell damage
  • Alcohol can also enhance the effects of the toxic metabolites of drugs e.g. paracetamol on the liver
69
Q

How do calculate alcohol units?

A

Alcohol units:

One unit of alcohol is equal to 10 mL of pure ethanol. The ‘strength’ of an alcoholic drink is determined by the ‘alcohol by volume’ (ABV).
Examples of one unit of alcohol:

  • 25ml single measure of spirits (ABV 40%)
  • a third of a pint of beer (ABV 5 to 6%)
  • half a 175ml ‘standard’ glass of red wine (ABV 12%)

To calculate the number of units in a drink multiply the number of millilitres by the ABV and divide by 1,000. For example:

  • half a 175ml ‘standard’ glass of red wine = 87.5 * 12 / 1000 = 1.05 units
  • one bottle of wine = 750 * 12 / 1000 = 9 units
  • one pint of 5% beer or lager = 568 * 5 / 1000 = 2.8 units
70
Q

What is alcoholic liver disease?

A

ALCOHOLIC LIVER DISEASE

  • A spectrum of alcohol-induced liver dysfunction ranging from mild, reversible fatty liver to irreversible liver fibrosis and cirrhosis.
  • It covers a spectrum of conditions:
    • alcoholic fatty liver disease
    • alcoholic hepatitis
    • cirrhosis

Epidemiology:

  • Common – difficult to say exactly how much as many cases don’t present or are asymptomatic
  • About 20% of alcoholics will get cirrhosis
  • Risk factors:
    • Continuous high alcohol intake
    • Binge drinking (although occasional binges better than drinking a lot every day)
    • Genetic predisposition
      • 50% of heavy drinkers have normal livers
    • Women most susceptible than men
71
Q

What is the pathophysiology of alcoholic liver disease?

A

Pathophysiology:

  • Alcohol processing in the liver increases the NAD/NADH ratio, which causes more fatty acid synthesis and less fatty acid oxidation, causing fatty liver.
  • Acetaldehyde produced from alcohol may also damage liver cells
  • Alcohol might convert normal hepatocytes into myofibroblasts, which lay down collagen and cause fibrosis.
  • Alcohol also enhances the effects of other toxins on the liver (e.g. paracetamol).
72
Q

Explain how excessive alcohol consumption can cause fatty liver, hepatitis and cirrhosis?

A

Fatty liver:

  • Metabolism of alcohol produces fat in the liver
  • This is minimal with small amounts of alcohol, but with larger amounts hepatocytes become accumulated with fat (steatosis)
  • There is no liver cell damage
  • The fat disappears on stopping alcohol
  • In some cases, collagen is laid down around the central hepatic veins and this can sometimes progress to cirrhosis without a preceding hepatitis
  • Alcohol directly affects stellate cells, transforming them into collagen- producing myofibroblast cells

Alcoholic hepatitis:

  • In addition to fatty change there is infiltration by polymorphonuclear leucocytes and hepatocyte necrosis
  • Dense cytoplasmic inclusions called Mallory bodies are sometimes seen in hepatocytes and giant mitochondria are also a feature
  • If alcohol consumption continues, alcoholic hepatitis can progress to Cirrhosis

Alcoholic cirrhosis:

  • Represents the final stage of liver disease from alcohol use
  • Classically of the MICRONODULAR TYPE but mixed pattern is also seen accompanying fatty change, and evidence of pre-existing alcoholic hepatitis may be present
73
Q

How does alcoholic hepatitis present?

A
  • Alcoholic Hepatitis
    • Cases range from mild with only derangement of liver function tests to severe with a high mortality rate.
    • Moderate cases may have mild jaundice, hepatomegaly and signs of chronic liver disease (ascites, bruising, clubbing, Dupuytren’s contracture)
    • Severe cases may have decreased GCS, encephalopathy, high bilirubin levels and prolonged prothrombin times. Along with abdominal pain, ascites, deep jaundice, hepatomegaly
74
Q

How does alcoholic cirrhosis present?

A
  • Cirrhosis
    • Patients can be very well with few symptoms
    • On examination there are usually signs of chronic liver disease - ascites, bruising, clubbing and Dupuytren’s contracture
    • There are features of alcohol dependency
75
Q

What investigations would you request for alcoholic liver disease?

A

Investigations:

  • Fatty liver
    • Ultrasound scan can pick up fatty change in the liver
    • Raised ALT and AST
    • High MCV due to drinking heavily
  • Alcoholic hepatitis
    • Patient with significant alcohol intake who develops deranged liver function tests; especially with an elevated bilirubin
    • AST:ALT usually greater than 2 – suggestive of acute alcoholic hepatitis
    • The transaminases are rarely > 500
    • Prolonged prothrombin time and ALP
    • gamma-GT is characteristically elevated
  • Alcoholic cirrhosis:
    • Same as alcoholic hepatitis
    • Can do liver biopsy or fibro-scan to confirm diagnosis
76
Q

What is the management of alcoholic liver disease?

A

Management:

  • Advise to cut down and ideally STOP drinking
    • Inpatient/outpatient detoxification schemes if appropriate

If admitted to hospital

  • Give intravenous B vitamins e.g. IV Pabrinex
    • Once discharged then give oral B vitamins
  • Monitor for and treat withdrawal symptoms e.g. Chlordiazepoxide PO
    • Use CIWA scoring system to see if they need benzos
  • Ensure adequate nutrition -> Give high-calorie supplements or NG feeding if needed
  • Avoid opiates
  • Give laxatives +/- enemas to ensure bowels opening 2-3 times a day

Fatty liver:

  • If patient stops alcohol then fat will disappear and things will go back to normal

Alcoholic hepatitis:

  • glucocorticoids (e.g. prednisolone) are often used during acute episodes of alcoholic hepatitis
    • Maddrey’s discriminant function (DF) is often used during acute episodes to determine who would benefit from glucocorticoid therapy
    • it is calculated by a formula using prothrombin time and bilirubin concentration
    • usually DF >32
  • OR pentoxyphylline (phosphodiesterase inhibitor) is also sometimes used
  • Mortality of severe alcoholic hepatitis is 50%

Alcoholic cirrhosis:

  • See above in CLD section
77
Q
A
78
Q

What is NALD?

A

NON-ALCOHOLIC FATTY LIVER DISEASE (NAFLD)

  • NAFLD is associated with obesity, abnormal glucose tolerance and dyslipidaemia
  • it has been described as the hepatic manifestation of the metabolic syndrome and hence insulin resistance is thought to be the key mechanism leading to steatosis.
  • It is now the most common cause of liver disease in the developed world.
  • Incidence of NAFLD and NASH is rising in children and adolescents
  • It is largely caused by obesity and describes a spectrum of disease ranging from:
    • steatosis - fat in the liver
    • steatohepatitis - fat with inflammation, non-alcoholic steatohepatitis (NASH)
    • progressive disease may cause fibrosis and liver cirrhosis
79
Q

What is Non-alcoholic steatohepatitis (NASH)?

A

Non-alcoholic steatohepatitis (NASH)

  • It is a term used to describe liver changes similar to those seen in alcoholic hepatitis in the absence of a history of alcohol abuse.
  • It is relatively common and thought to affect around 3-4% of the general population.
  • The progression of disease in patients with NASH may be responsible for a proportion of patients previously labelled as cryptogenic cirrhosis.
80
Q

How does NAFLD present?

A

Features:

  • usually asymptomatic if steatosis
  • hepatomegaly- very common
  • RUQ, fatigue, malaise if steatohepatitis
  • splenomegaly (with cirrhosis only)
  • signs of CLD (ascites, oedema, sider naevi, jaundice)- seen in patients with cirrhosis
  • ALT is typically greater than AST
  • increased echogenicity on ultrasound
81
Q

List 4 DD of NAFLD

A

Differentials:

  • Alpha 1-antitrypsin deficiency.
  • Autoimmune hepatitis.
  • Haemochromatosis
  • Drug-induced hepatotoxicity.
82
Q

What investigations would you request for NAFLD?

A

Investigations:

  • there is no evidence to support screening for NAFLD in adults, even in at risk groups (e.g. type 2 diabetes)
  • Bloods:
    • LFTs:
      • mildly raised ALT is the first change relative to AST but this tends to reverse if disease progresses, and then ALT falls.
      • Up to 50% of patients can have normal ALT and AST levels[
      • Further LFT changes if alcohol is the cause (raised (GGT)).
    • Fasting lipids – raised
    • Fasting glucose
    • FBC
    • Autoimmune studies (ANA, ASMA may be raised in NASH)
    • Check for other causes e.g. Iron studies, Ceruloplasmin, Viral studies (hepatitis)
  • Diagnosis of steatosis is made via USS:
    • Shows a hyper-echogenic, bright image
    • Steatosis tends to be focal whereas steatohepatitis is diffuse
  • in these patients with incidental finding of steatosis on USS - NICE recommends the use of the enhanced liver fibrosis (ELF) blood test to check for advanced fibrosis
  • the ELF blood test is a combination of hyaluronic acid + procollagen III + tissue inhibitor of metalloproteinase 1.
  • if the ELF blood test was not available:
    • non-invasive tests may be used to assess the severity of fibrosis
    • these include the FIB4 score or NALFD fibrosis score
    • these scores may be used in combination with a FibroScan (liver stiffness measurement assessed with transient elastography)
    • this combination has been shown to have excellent accuracy in predicting fibrosis
  • liver biopsy is the only definitive test to confirm diagnosis and exclude other causes – done ONLY AFTER having the tests that predict there is likely advanced fibrosis in order to stage the disease accurately
83
Q

What is the management of NAFLD?

A

Management:

  • the mainstay of treatment is monitoring and lifestyle changes particularly weight loss, with low fat diabetes diet, regular exercise and alcohol abstinence.
  • There are currently no drugs licensed for NASH in the UK
  • there is ongoing research into the role of gastric banding and insulin-sensitising drugs (e.g. metformin, pioglitazone)
84
Q

Explain the causes of drug induced liver damage

A

DRUG INDUCED LIVER INJURY:

  • Liver is the major site of drug metabolism
  • Drugs are converted from fat-soluble to water-soluble substances that can be excreted in the urine or bile
  • Mechanisms involved that cause liver damage by drugs:
    • Disruption of intracellular Ca2+ homeostasis
    • Disruption of bile canalicular transport mechanisms
    • Induction of apoptosis
    • Inhibition of mitochondrial function, which prevents fatty acid metabolism and accumulation of both lactate and reactive oxygen species
  • Most drug reactions occur within 3 months of starting drug
  • Onset usually seen 1-12 weeks of starting (earlier is unusual)
  • Damage may occur several weeks after stopping drug
  • Monitoring liver biochemistry in patients on long-term treatment, such as anti-tuberculosis therapy is ESSENTIAL
  • If a drug is suspected of causing hepatic damage it should be STOPPED IMMEDIATELY
  • Main causes:
    • Antibiotics (30-40%):
      • Co-amoxiclav, Flucloxacillin, TB drugs and Erythromycin
    • CNS drugs (15%):
      • Chlorpromazine, Carbamazepine
    • Immunosuppressants (5%)
    • Analgesics (20%) - DICLOFENAC
    • GI drugs (10%) e.g. PPIs
  • Drugs that do not cause injury:
    • Low-dose aspirin
    • NSAIDs EXCEPT diclofenac
    • Beta blockers
    • Ace inhibitors
    • Thiazides
    • Calcium channel blockers
  • Usually (90%) there is resolution within 3 months of stopping drug
  • In 10% there is prolonged injury that can result in long term damage

Drug-induced liver disease is generally divided into hepatocellular, cholestatic or mixed.

The following drugs tend to cause a hepatocellular picture:

  • paracetamol
  • sodium valproate, phenytoin
  • MAOIs
  • halothane
  • anti-tuberculosis: isoniazid, rifampicin, pyrazinamide
  • statins
  • alcohol
  • amiodarone
  • methyldopa
  • nitrofurantoin

The following drugs tend to cause cholestasis (+/- hepatitis):

  • combined oral contraceptive pill
  • antibiotics: flucloxacillin, co-amoxiclav, erythromycin*
  • anabolic steroids, testosterones
  • phenothiazines: chlorpromazine, prochlorperazine
  • sulphonylureas
  • fibrates
  • rare reported causes: nifedipine

The following drugs tend to cause liver cirrhosis

  • methotrexate
  • methyldopa
  • amiodarone
85
Q

What is decompensated liver disease?

A

DECOMPENSATED CHRONIC LIVER DISEASE

It is when patients with chronic liver disease can present with acute decompensation due to various causes. The decompensation may take the form of any of the following complications:

· Oesophageal variceal bleed

· Ascites

· Spontaneous bacterial peritonitis

· Hepatic encephalopathy

· Hepatorenal syndrome

· Hepatocellular carcinoma

86
Q

What is portal hypertension and varices?

List the pre-hepatic, hepatic and post hepatic causes

A

PORTAL HYPERTENSION AND VARICES

  • Portal vein is formed by the union of the superior mesenteric and splenic veins
  • Normal pressure is 5-8mmHg with only a small gradient across the liver to the hepatic vein, in which blood is returned to the heart via the inferior vena cava
  • Oesophageal varices are dilated oesophageal veins secondary to portal hypertension.
    • Responsible for 5% of episodes of GI bleeding in the UK.

Aetiology:

  • Pre-hepatic
    • Portal vein thrombosis / obstruction
    • Increased portal blood flow: fistula
  • Hepatic
    • Cirrhosis
      • 90% of cirrhotic patients get varices, 30% bleed
    • Acute hepatitis (esp. alcoholic)
    • Schistosomiasis
    • Congenital hepatic fibrosis
  • Post-hepatic
    • Compression (e.g. from tumour)
    • Budd-Chiari syndrome
    • Constrictive pericarditis (and rarely right-sided heart failure)
87
Q

Explain the pathophysiology of portal hypertension

A

Pathophysiology:

  • Following liver injury and fibrogenesis e.g due to cirrhosis, the contraction of activated myofibroblasts (mediated by endothelin, nitric oxide and prostaglandins) contributes to increased resistance to blood flow
  • This leads to portal hypertension → splanchnic vasodilation → drop in BP → increased cardiac output to compensate for BP → salt and water retention to increase blood volume and compensate → hyperdynamic circulation (high circulatory volume)/increased portal flow → Formation of collaterals between the portal and systemic systems e.g. in the lower oesophagus and gastric cardia
  • Sites of collaterals/varices:
    • Gastro-oesophageal junction - superficial and tend to rupture
  • Rectum (30%)
  • Left renal vein
  • Diaphragm
  • Retroperitoneum
  • Microvasculature of the gut becomes congested and gives rise to portal hypertensive gastropathies and colonopathies
88
Q

What are the clinical features of PHT and variceal bleeds?

A

Clinical features:

  • Patients are often asymptomatic
  • Only clinical sign being splenomegaly which is unspecific
  • Present with variceal bleeding
89
Q

What is the management of PHT/variceal bleeding?

A

Management of variceal bleeds:

  • Resuscitation
    • ABC

§ Oxygen, blood tests (VBG, FBC, U+Es, LFTs, clotting, X-match)

§ Erect CXR

§ Fluid resuscitation

§ HDU/ITU

§ Monitoring

  • CVP line and catheter
  • Correct anaemia and coagulopathy
    • Transfusion trigger should be 7 (aim 7-9)
    • Give vitamin K and platelet infusion for clotting abnormalities
  • Terlipressin IV
    • Vasopressin analogue. Reduces portal pressure. Contraindicated in shock and peripheral vascular disease.
    • Octreotide (a somatostatin analogue) can also be used second line
  • Antibiotics
    • Broad spectrum. IV Tazocin
  • Endoscopy (once stable and not bleeding)
  • Band ligation

§ This is the first choice of treatment

  • Sclerotherapy

§ In this therapy the varices are sclerosed

§ Various sclerosants can be used

§ Complications include transient fever, dysphagia, chest pain, ulceration and stricture.

  • Variceal obturation with glue

§ This involves embolisation of varices with a glue-like substance

§ Particularly good for gastric or gastro-oesophageal variceal bleeding

§ However, there is a risk of embolisation to the lung, spleen or brain

  • Transjugular intrahepatic portosystemic shunt (TIPSS)
    • Where bleeding is not controlled by endoscopy
    • Patient needs to be transferred to a specialist liver unit
    • Hepatic vein is cannulated percutaneously via the internal jugular vein using a needle under ultrasound guidance and a tract is created through the liver from the hepatic to the portal vein reducing portal pressure.
    • High success rate but encephalopathy found in 25% cases (as portal blood diverted from the liver) and shunt occludes within 1 year in up to 50% cases
90
Q

What is the prevention of PHT/ variceal bleeding?

A

Prevention of variceal bleeding:

  • Beta blockers e.g. PROPRANOLOL
    • These lower portal blood pressure and risk of further bleeding by reducing portal blood flow.
  • Nitrates
    • Just for secondary prophylaxis.
    • Nitrates can also be used in the acute variceal haemorrhage with vasopressin and terlipressin.
  • Endoscopic screening
    • All patients with newly-diagnosed cirrhosis should have screening endoscopy, looking for oesophageal varices. In the long-term, repeated endoscopic screening is usually required, e.g. 2 to 3-yearly in cases of small varices.
91
Q

What is ascites?

List some causes

A
  • Ascites is the accumulation of free fluid within the peritoneal cavity
  • The causes of ascites can be grouped into those with a serum-ascites albumin gradient (SAAG) <11 g/L or a gradient >11g/L as per the table below
  • SAAG is SERUM ALBUMIN MINUS ASCITIC ALBUMIN
  • If low (<11g/l [or 1.1g/dl]) = non-portal hypertensive (sort of =) exudate
  • High=portal hypertensive = fluid leaking into peritoneum
92
Q

Explain how low protien and low fluid flow can cause ascited

A
  • Low protein (low ALBUMIN there is an inability to pull fluid back into the intravascular space) e.g. nephrotic syndrome, hypoalbuminemia, malnutrition
  • Low fluid flow:
    • Fluid cannot move forwards through system e.g. due to a clot
    • Raises pressure in vessels causing fluid to leak out of the vessels
    • E.g.:
      • Cirrhosis - COMMONEST CAUSE
      • Budd-chiari syndrome (occlusion of hepatic veins that drain
      • liver)
      • Cardiac failure
      • Constrictive pericarditis
93
Q

How does ascites present?

A

Clinical features:

  • Abdominal swelling may develop over days or several weeks - Distended abdomen
  • Fullness in the flanks and SHIFTING DULLNESS
  • Mild abdominal pain and discomfort are common
  • If severe pain then raises suspicion of bacterial peritonitis
  • Respiratory distress and difficulty eating accompany tense ascites
  • May be scratch marks on abdomen caused by itching due to jaundice i.e. liver failure
  • Many patients also have peripheral oedema
94
Q

What investigations would you request for ascites?

A

Investigations:

  • Presence of fluid is confirmed by demonstrating shifting dullness
  • Bloods: FBC (anaemia and platelets), U&E, LFT, clotting, gas for pH
  • Abdominal USS:
    • Liver mets, hepatic vein and artery Doppler, splenomegaly (portal HTN)
  • Chest x-ray may show pleural effusion or HF
  • Diagnostic paracentesis of 10-20ml of fluid using ascitic tap for:
  • Protein or albumin
  • white cell count – raised is indicative of bacterial peritonitis
  • culture and sensitivity
  • Cytology to find malignancy
  • Amylase to exclude pancreatic ascites (if >2000 is pancraetitis)
  • Protein measurement of ascitic fluid from ascitic tap:
  • Transudate (less bad) - low protein (< 30g/L) - transparent i.e. no/little protein:
  • Portal hypertension e.g. cirrhosis - Constrictive pericarditis
  • Cardiac failure
  • Budd-Chiari syndrome
  • Exudate (extremely bad) - high protein (> 30g/L) - exudes protein:
  • Malignancy
  • Peritonitis
  • Pancreatitis
  • Peritoneal tuberculosis
  • Nephrotic syndrome
95
Q

What is the management of ascites?

A

Management:

  • reducing dietary sodium (<5.2 g)
  • fluid restriction is sometimes recommended if the sodium is < 125 mmol/L
  • Avoid alcohol
  • Diuretic of choice is aldosterone antagonists: e.g. SPIRONOLACTONE
  • loop diuretics are often added
  • therapeutic abdominal paracentesis used for tense ascites
  • can drain 5L at a time
  • Human albumin solution (20%) should be given alongside all large-volume paracentesis where more than 5L is removed
  • paracentesis induced circulatory dysfunction can occur due to large volume paracentesis (> 5 litres).
  • It is associated with a high rate of ascites recurrence, development of hepatorenal syndrome, dilutional hyponatraemia, and high mortality rate
  • prophylactic antibiotics to reduce the risk of spontaneous bacterial peritonitis.
    • Nice recommends ORAL CIPROFLOXACIN OR NORFLOXACIN for people with cirrhosis and ascites with an ascitic protein of 15 g/litre or less, until the ascites has resolved
  • a transjugular intrahepatic portosystemic shunt (TIPS) may be considered in some patients with refractory ascites needing frequent paracentesis (>3/month).
96
Q

What is the prognosis of patients with ascites?

A

Prognosis:

  • 50% mortality over two years, and signifies the need to consider liver transplantation.
  • Refractory ascites carries an even poorer prognosis, 50% patients dying within six months.
  • Therapeutic paracentesis and TIPSS do not improve long term survival
97
Q

What is pertionitis?

A

PERITONITIS

  • It is inflammation of the peritonitis

Physiology:

  • Consists of two parts:
  • Parietal:
    • Covers the abdominal wall
    • SOMATIC INNERVATION
    • Sensation is well localised
  • Visceral:
    • On organs e.g. stomach, liver and colon
    • AUTONOMIC INNERVATION
    • Sensation is poorly localised
  • Peritoneal cavity is a closed sac lined by mesothelial cells; these produce surfactant, which acts as a lubricant within the peritoneal cavity
  • The cavity contains < 100mL of serous fluid containing < 30g/L of protein (transudate)
  • The mesothelial cells lining the diaphragm have gaps that allow communication between the peritoneum and the diaphragmatic lymphatics
  • Around 1/3rd of fluid drains through these lymphatics, the remainder through the parietal peritoneum
  • These mechanisms allow particulate matter to be removed rapidly from the peritoneal cavity
98
Q

Where will you feel the pain if pertionitis occurs in the foregut, midgut or hindgut?

A
99
Q

What are the classifications of peritonitis?

A

Classification:

  • Primary peritonitis:
    • It is inflammation on its own:
      • Spontaneous Bacterial peritonitis
      • Ascites
    • Diagnosis is an ascitic tap and blood cultures
    • Treatment is with broad spectrum antibiotics
  • Secondary peritonitis:
    • Caused by something else e.g. chemical i.e. bile
    • Treat the cause, primarily via surgery
100
Q

What are the features of peritonitis?

A

Features:

  • Poorly localised pain (irritation of visceral peritoneum) then moving to one point on abdomen and becoming localised (when it begins to irritate the parietal peritoneum)
  • Perforation =SUDDEN ONSET acute severe abdominal pain followed by general collapse and shock
  • When peritonitis is secondary to inflammatory disease, the onset is less rapid with the initial features being those of the underlying disease
  • Lying still - people with peritonitis want to stay still
  • Speedbumps are very painful for people with peritonitis
  • Pain relieved by resting hands on abdomen - thereby stopping movement of peritoneum and thus pain
  • Rigid abdomen
101
Q

What investigations would you request for peritonitis?

A

Investigations:

  • Erect CXR to check for air under the diaphragm
  • Serum amylase to rule out pancreatitis.
  • Ultrasound / CT to confirm diagnosis
102
Q

What are the complications of peritonitis?

A

Complications:

  • Loss of fluids / disturbance of electrolyte balance – hypovolaemia may result and this could bring about shock or even renal failure.
  • Difficulty breathing – due to pressure of fluid on diaphragm.
  • Peritoneal abscess:
    • should be suspected if a patient continues to remain unwell post-op, with a swinging fever, high white cell count and continuing pain
  • Toxaemia and septicaemia - if there is any delay in treatment can produce a worse.

Abscesses - commonly pelvic or subphrenic

103
Q

What is the management of peritonitis?

A

Management:

  • IV fluids and electrolytes are given to try and reverse hypovolemia and restore the normal electrolyte balance.
  • IV antibiotics may be given if it is an infective peritonitis.
  • Surgery – a laparotomy and peritoneal lavage of abdominal cavity
  • If left untreated, peritonitis is almost always fatal.
104
Q

What is SBP?

A

SPONTANEOUS BACTERIAL PERITONITIS (SBP)

  • Spontaneous bacterial peritonitis (SBP) is a form of peritonitis usually seen in patients with ascites secondary to liver cirrhosis.
  • 10-30% of patients with ascites and has mortality rate of 20%.
  • Organisms are usually E. coli, streptococci and enterococci.
  • Alcoholic liver disease is a marker of poor prognosis in SBP.
105
Q

How does SBP present?

A

Clinical Features

  • Generalised abdominal pain
  • Hepatic encephalopathy, renal impairment or peripheral leucocytosis without any obvious precipitating
  • ascites
  • fever
106
Q

How is SBP diagnosed?

A

Diagnosis

  • paracentesis: neutrophil count > 250 cells/ul
  • the most common organism found on ascitic fluid culture is E. coli
107
Q

How is SBP managed?

A

Management

  • intravenous cefotaxime +/- metronidazole
  • If fluid resuscitation needed for septic shock then try to avoid colloid/crystalloid and use plasma expander such as human albumin solution instead.
108
Q

What is primary biliary Cirrhosis?

A

PRIMARY BILIARY CIRRHOSIS

  • An autoimmune condition in which progressive destruction of the bile ducts eventually leads to cirrhosis.
  • The classic presentation is itching in a middle-aged woman
  • Also known as Primary biliary cholangitis

Epidemiology

  • Prevalence ≤ 4 in 100,000
  • Females: males = 9:1 A LOT MORE COMMON IN FEMALES
  • Peak incidence around 50 years
  • Commonest in northern Europeans, least common in Africans.
109
Q

What are the causes of primary biliary cirrhosis and describe its pathophysiology

A

Aetiology:

  • A combination of genetic predisposition and environmental triggers
  • Various studies have suggested an association with urinary tract infections, cigarette smoking and reproductive hormone use.

Pathophysiology:

  • Interlobar bile ducts are damaged by CHRONIC AUTOIMMUNE GRANULOMATOUS INFLAMMATION resulting in cholestasis which may lead to fibrosis, cirrhosis and portal hypertension
  • Serum anti-mitochondrial antibodies (AMA) found in almost all patients
110
Q

List 4 clinical features and 4 signs of primary biliary cirrhosis

A

Clinical features:

  • Asymptomatic (often diagnosed after incidental finding of abnormal liver function tests i.e. Raised ALP)
  • Fatigue and lethargy
  • Pruritus
  • Abnormal bleeding or bruising
  • Pale stool or dark urine
  • around 10% of patients have right upper quadrant pain

Signs:

  • Cholestatic Jaundice
  • Clubbing
  • Xanthelasma and tendon xanthomata
  • Hepatosplenomegaly
  • hyperpigmentation, especially over pressure points
  • Features of chronic liver disease
    • Palmar erythema; Dupuytren’s contracture; gynaecomastia; spider naevi; cachexia; ascites
  • Ecchymoses
  • Features of other autoimmune diseases (e.g. rheumatoid arthritis, diabetes, thyroid disorders, hypoadrenalism, vitiligo)
  • late: may progress to liver failure
111
Q

What conditions are associated with primary biliary cirrhosis?

A

Associations:

  • Sjogren’s syndrome (seen in up to 80% of patients)
  • rheumatoid arthritis
  • systemic sclerosis
  • thyroid disease (hypothryodism)
112
Q

What are the complications of primary biliary cirrhosis?

A

Complications:

  • malabsorption of fats and fat-soluble vitamins which can lead to osteomalacia, coagulopathy
  • osteoporosis
  • sicca syndrome occurs in 70% of cases
  • Cirrhosis and its complications e.g. portal hypertension: ascites, variceal haemorrhage
  • hepatocellular cancer (20-fold increased risk)
113
Q

What investigations would you request for primary biliary cirrhosis?

A

Investigations:

  • Blood tests:
    • Liver function tests
    • Raised ALP and GGT with mildly elevated transaminases initially.
    • In later disease the bilirubin starts to rise.
  • Clotting
    • Elevated prothrombin time
  • Full blood count
    • Thrombocytopenia if cirrhosis present
  • Serum lipids
    • Cholesterol, LDL and HDL all significantly raised
  • Full liver screen of blood tests to rule out other causes of liver disease
  • Autoimmune screen:
    • anti-mitochondrial antibodies (AMA) M2 subtype are present in 98% of patients and are highly specific
    • smooth muscle antibodies in 30% of patients
    • raised serum IgM
  • Ultrasound liver to look for focal liver lesions, portal/hepatic vein thrombosis, extrinsic causes of biliary duct compression.
  • MRCP gives a more detailed view of the biliary tree and does not have the associated morbidity of ERCP
  • Liver biopsy shows characteristic histological features:
  • Portal tract infiltrate, mainly of lymphocytes and plasma cells
  • Around 40% have granulomas
  • Damage to and loss of small bile ducts and ductular proliferation
  • Portal tract fibrosis and eventually cirrhosis is seen
114
Q

What is the diagnostic criteria of PBC?

A

Recommended diagnostic criteria

A diagnosis of PBC can be made if two of the following three criteria are met:

  • Biochemical evidence of cholestasis (i.e. elevation of alkaline phosphatase)
  • Presence of antimitochondrial antibodies
  • Histology showing non-suppurative cholangitis and destruction of interlobular bile ducts
115
Q

What is the management of PBC?

A

Management:

  • For pruritus:
    • CHOLESTYRAMINE
    • NALOXONE or NALTREXONE (opioid antagonists) shown to help
  • fat-soluble vitamin supplementation (ADEK)
  • Bisphosphonates for patients at high risk of osteoporosis or T score < 2.5
  • URSODEOXYCHOLIC ACID:
    • Can significantly improve liver biochemistry (bilirubin and aminotransferases) and reduce the need for liver transplantation and overall mortality.
    • However, has no effect on pruritus, fatigue or associated bone disease
  • liver transplantation e.g. if bilirubin > 100 (PBC is a major indication) - recurrence in graft can occur but is not usually a problem
  • The serum bilirubin level is a marker of prognosis
116
Q

What is primary sclerosising cholangitis?

A

PRIMARY SCLEROSING CHOLANGITIS

  • It is a biliary disease of unknown aetiology characterised by inflammation and fibrosis of intra and extra-hepatic bile ducts.

Epidemiology:

  • Male:Female = 2:1
  • Usually presents from 20-30 years old onward but can present earlier
  • Studies show a prevalence varying from 0.2 to 8.5 per 100,000
  • Higher prevalence in areas where ulcerative colitis is more common
    • 4% of patients with UC have PSC, 80% of patients with PSC have UC
  • Other associations are crohns (less commonly) and HIV
117
Q

What are the causes of PSC?

A
118
Q

What are the clinical features of PSC?

A

Clinical Features:

  • Fatigue
  • Pruritus
  • Weight loss
  • Right upper quadrant (RUQ) pain
  • Recurrent biliary infections

Signs:

  • Jaundice – cholestatic
  • Hepatomegaly
  • Features of chronic liver disease
    • Dupuytren’s contracture; palmar erythema; gynaecomastia; spider naevi; ascites; cachexia
  • Features of IBD
    • Stoma; abdominal scars; mouth ulcers; erythema nodosum
119
Q

What is the management of PSC?

A

Management:

  • Anti-pruritics: as per management in PBC
  • Vitamin A,D,E and K replacement
    • Vitamin D and Calcium replacement
  • If bacterial cholangitis
    • Antibiotics plus therapeutic drainage of the obstruction
    • Sometimes patients require long-term prophylactic antibiotics if they are suffering recurrent infections
  • Stenting/drainage of strictures
    • Endoscopic balloon dilatation, sphincterotomy and stent placement should be performed initially.
    • If endoscopic therapy is unsuccessful then biliary dilatation and/or stent placement should be attempted percutaneously.
  • liver transplant
    • Transplantation criteria are the same for PSC patients as for all patients with chronic liver disease and also include recurrent cholangitis, intractable pruritus and limited cholangiocarcinoma.
    • Outcomes are good with a 5-year survival rate of 85%.
120
Q

What are the complications of PSC?

A

Complications

  • Biliary infections
  • Cirrhosis and associated complications
  • Cholangiocarcinoma (in 10%)
    • Any PSC patient whose liver biochemistry deteriorates should be investigated for a possible underlying cholangiocarcinoma.
  • Colorectal carcinoma (increased risk)
    • should undergo colonoscopic surveillance 1-2 yearly from PSC diagnosis
121
Q

What is a volvulus?

A
  • Volvulus may be defined as torsion of the colon around it’s mesenteric axis resulting in compromised blood flow and closed loop obstruction.
  • Can occur at stomach, small intestine, caecum, transverse colon and sigmoid colon
  • Sigmoid volvulus (around 80% of cases i.e. commonest form) describes large bowel obstruction caused by the sigmoid colon twisting on the sigmoid mesocolon.
  • A similar problem may also occur at the caecum (20% of cases).
  • In most people (around 80%) the caecum is a retroperitoneal structure so not at risk of twisting.
  • In the remaining minority there is developmental failure of peritoneal fixation of the proximal bowel putting these patients at risk of caecal volvulus.
122
Q

Describe the embryology involved in the formation of the GI tract

A

Embryology:

  • 4th week of gestation the GI tract is a straight line
  • At 8th week the midgut (D2 to 2/3rds transverse colon)
  • rotates and become fixed to the posterior wall
  • Any arrest in the development at any stage will result in the narrowing of the mesenteric base and impairs fixation onto posterior abdominal wall thereby increasing risk of volvulus
123
Q

What are the clinical features of a volvulus?

A

Clinical features

  • constipation
  • abdominal bloating
  • abdominal pain
  • nausea/vomiting (bilious vomiting)
  • anorexia
  • failure to thrive in babies

Volvulus of stomach (rare) syx:

  • Classical triad of GI obstruction features:
  • Vomiting (then retching)
  • Pain
  • Failed attempts to pass a NG tube
  • Regurgitation of saliva also occurs
  • Dysphagia and noisy gastric peristalsis (relieved by lying down) may occur in chronic volvulus
124
Q

What are associated with sigmoid volvulus?

A
  • older patients
  • chronic constipation
  • Chagas disease
  • neurological conditions e.g. Parkinson’s disease, Duchenne muscular dystrophy
  • psychiatric conditions e.g. schizophrenia
125
Q

What are associated with caecal volvulus?

A
  • all ages
  • adhesions

pregnancy

126
Q

List 3 dd of a volvulus?

A

Differential diagnosis:

  • Other causes of acute obstruction
  • Appendicitis, cholecystitis, constipation, gastroenteritis, GORD, hepatitis, peptic ulcer and pancreatitis
127
Q

What investigations would you request for a volvulus?

A

Investigations:

  • usually diagnosed on AXR
    • sigmoid volvulus: large bowel obstruction (large, dilated loop of colon, often with air-fluid levels) + coffee bean sign
    • midgut volvulus: there is partial duodenal obstruction (dilation of both stomach and proximal duodenum, with a small amount of distal bowel gas) – this is double bubble sign
    • caecal volvulus: small bowel obstruction may be seen
128
Q

What is the management of a volvulus?

A

Management:

  • Non surgical: GI compression with NG tube
  • Surgical correction e.g. Ladd’s procedure (rotating small intestine in anticlockwise direction) is the treatment of choice
  • For sigmoid volvulus: decompress using rigid sigmoidoscopy with rectal tube insertion, if this fails then surgery which is resection of the redundant sigmoid colon
  • For caecal volvulus: management is usually operative. Right hemicolectomy is often needed
129
Q

What are the complications of a volvulus?

A

Complications:

  • Intestinal ischaemia, mucosal necrosis and sepsis
  • Perforation and peritonitis = can be fatal
  • Chronic intermittent volvulus leads to malabsorption
  • Sigmoid volvulus ca lead to bowel obstruction
  • Postoperatively, the main complication is short-gut syndrome which can lead to growth restriction
130
Q

What is haemochromatosis?

Who does it more commonly affect?

List 5 clinical features

Which complications are reversible and irreversible?

Describe the pathophysiology

What investigations would you request and expect to see?

What is the management?

A

HAEMOCHROMATOSIS

  • It is an autosomal recessive disorder of iron absorption and metabolism resulting in iron accumulation n joints, liver, heart, pancreas, pituitary, adrenals and skin
  • It is caused by inheritance of mutations in the HFE gene on both copies of chromosome 6.
  • RF: Family history, Alcoholics
  • It is often asymptomatic in early disease and initial symptoms often non-specific e.g. lethargy and arthralgia

Epidemiology

  • 1 in 10 people of European descent carry a mutation in the genes affecting iron metabolism, mainly HFE
  • prevalence in people of European descent = 1 in 200, making it more common than cystic fibrosis
  • Most common single gene disorder in Caucasians
  • More common in MALES (esp. middle aged men) than females since menstrual blood loss is protective
  • Most present in their 50s

Presenting features

  • early symptoms include fatigue, erectile dysfunction and arthralgia (often of the hands)
  • ‘bronze’ skin pigmentation
  • diabetes mellitus
  • liver: stigmata of chronic liver disease, hepatomegaly, cirrhosis, hepatocellular deposition)
  • cardiac failure (2nd to dilated cardiomyopathy) – common esp. in younger patients.
  • hypogonadism i.e. less testosterone and oestrogen (2nd to cirrhosis and pituitary dysfunction - hypogonadotrophic hypogonadism)
  • arthritis (especially of the hands)
  • Joint x-rays characteristically show chondrocalcinosis

Reversible complications-

  • Cardiomyopathy
  • Skin pigmentation

Irreversible complications-

  • Liver cirrhosis**
  • Diabetes mellitus
  • Hypogonadotrophic hypogonadism
  • Arthropathy

Pathophysiology:

  • The HFE gene protein interacts with the transferrin receptor 1, which is a mediator in intestinal iron absorption
  • Iron is taken up by the mucosal cells of the small intestine inappropriately, EXCEEDING the binding capacity of transferrin
  • Hepcidin, a protein synthesise in the liver, is central to the control of iron absorption; it is increased in iron deficiency states and decreased with iron overload
  • Hepatic expression of the hepcidin gene is decreased in HFE haemochromatosis thereby facilitating iron overload
  • Excess iron is then gradually taken up by the liver and other tissue over a long period
  • The iron itself precipitates fibrosis
  • In symptomatic patients, the total body iron content is 20-40g (compared
  • with 3-4g in a normal person)
  • The iron content is particularly increased in the liver and pancreas but also in other organs e.g. heart, skin and endocrine glands
  • In established cases, the liver shows extensive iron deposition and fibrosis
  • Cirrhosis is a late feature

Investigations:

  • general population:
    • transferrin saturation - most useful marker
    • Ferritin should also be measured but is not usually abnormal in the early stages of iron accumulation
    • Serum iron
  • Diagnostic tests
  • molecular genetic testing for the C282Y and H63D mutations
  • liver biopsy: Perl’s stain
  • Can also do ECHO to check if there is cardiomyopathy

Typical iron study profile in patient with haemochromatosis

  • transferrin saturation > 55% in men or > 50% in women
  • raised ferritin (e.g. > 500 ug/l) and iron
  • low TIBC

Management:

  • Venesection is the first-line treatment
    • Regular removal of blood, then body can use some of the excess iron to make new RBCs - effective at reducing iron excess
    • Required lifelong - done 3-4 times a year
    • monitoring adequacy of venesection: transferrin saturation should be kept below 50% and the serum ferritin concentration below 50 ug/l
  • DESFERRIOXAMINE may be used second-line in those that cannot tolerate venesection due to severe cardiac disease or anaemia
  • Diet low in iron - tea, coffee or red wine will reduce iron absorption
  • Avoid fruit/fruit juice (high in vitamin C) as these increase iron absorption
  • Screening:
    • All first-degree relatives should be screened to detect early and asymptomatic disease
131
Q

What is wilson’s disease?

Who does it more commonly affect?

List 5 clinical features

Describe the pathophysiology

What investigations would you request and expect to see?

What is the management?

A

WILSONS DISEASE

  • Wilson’s disease is an autosomal recessive disorder characterised by excessive copper deposition in the tissues.
  • Metabolic abnormalities include increased copper absorption from the small intestine and decreased hepatic copper excretion.
  • It is caused by a defect in the ATP7B gene located on chromosome 13 resulting in a molecular defect within a copper-transporting ATPase
  • The onset of symptoms is usually between 10 - 25 years.
  • Children usually present with liver disease whereas the first sign of disease in young adults is often neurological disease

Epidemiology:

  • Rare inherited disorder of biliary copper excretion with too much copper in the liver and CNS (basal ganglia i.e. globus pallidus and putamen)
  • More common in Caucasians than in Indians and Asians
  • Occurs worldwide especially in countries where marrying first degree relatives is common (consanguinity)

Pathophysiology:

  • Dietary copper is normally absorbed from the stomach and upper small intestine
  • Copper is transported to the liver loosely bound to albumin Once in the liver it is incorporated into a glycoprotein called
  • CAERULOPLASMIN synthesised in the liver and secreted into the blood
    The remaining copper is normally excreted in the bile and excreted in faeces
  • Wilson’s disease is a very rare inborn error of copper metabolism that results in copper deposition in various organs, including the liver, the basal ganglia of the brain and the cornea
  • The precise mechanism for the FAILURE OF COPPER EXCRETION IS UNKNOWN
  • Liver histology is not diagnostic but varies from that of chronic hepatitis to macronodular cirrhosis
  • The basal ganglia are damaged and show cavitation, the kidneys show tubular degeneration and erosions are seen in the bones
  • It is potentially treatable and all young patients with liver disease must be screened for this condition

Clinical presentation:
Features result from excessive copper deposition in the tissues, especially the brain, liver and cornea:

  • liver: hepatitis, cirrhosis
  • neurological:
    • basal ganglia degeneration: in the brain, most copper is deposited in the basal ganglia, particularly in the putamen and globus pallidus
    • speech, behavioural and psychiatric problems are often the first manifestations
    • also: asterixis, chorea, dementia, parkinsonism

Kayser-Fleischer rings

  • green-brown rings in the periphery of the iris
  • due to copper accumulation in Descemet membrane
  • present in around 50% of patients with isolated hepatic Wilson’s disease and 90% who have neurological involvement
  • renal tubular acidosis (esp. Fanconi syndrome)
  • haemolysis
  • blue nails

Investigations:

  • slit lamp examination for Kayser-Fleischer rings
  • reduced serum caeruloplasmin
  • reduced total serum copper (counter-intuitive, but 95% of plasma copper is carried by ceruloplasmin)
    • free (non-ceruloplasmin-bound) serum copper is increased
  • increased 24hr urinary copper excretion
  • Liver biopsy shows increased hepatic copper, hepatitis and cirrhosis - but note high copper levels are also found in chronic cholestasis
  • Haemolysis and anaemia may be present
  • MRI will show basal ganglia and cerebellar degeneration

Management

  • Avoid foods that are high in copper e.g. liver, chocolate, nuts, mushrooms and shellfish
  • Life- long PENICILLAMINE (chelates copper) - first-line treatment
  • Side effects:
  • Skin rashes
  • Fall in white cell count, Hb and platelets
  • Haematuria
  • Renal damage
  • trientine hydrochloride is an alternative chelating agent which may become first-line treatment in the future
  • Liver transplant if severe disease
  • Screen siblings as asymptomatic homozygotes need treating
132
Q

What is ALPHA1 -ANTITRYPSIN DEFICIENCY?

Who does it more commonly affect?

List 5 clinical features

Describe the pathophysiology

What investigations would you request and expect to see?

What is the management?

A

ALPHA1 -ANTITRYPSIN DEFICIENCY

  • It is a common inherited condition caused by a lack of a protease inhibitor (Pi) normally produced by the liver.
  • The role of A1AT is to protect cells from proteolytic enzymes such as NEUTROPHIL ELASTASE.
  • This deficiency affects the lung (emphysema) and liver (cirrhosis and hepatocellular cancer) and can present in homozygous and heterozygous forms
  • More common in CAUCASIANS
  • It classically causes emphysema (i.e. chronic obstructive pulmonary disease) in patients who are young and non-smokers.
  • Genetics
  • located on chromosome 14
  • inherited in an autosomal recessive / co-dominant fashion*
  • alleles classified by their electrophoretic mobility - M for normal, S for slow, and Z for very slow
  • normal = PiMM
  • homozygous PiSS (50% normal A1AT levels)
  • homozygous PiZZ (10% normal A1AT levels)

Features:

  • patients who manifest disease usually have PiZZ genotype
  • Respiratory (75%): panacinar emphysema, most marked in lower lobes
  • Hepatic: cirrhosis and hepatocellular carcinoma in adults, cholestasis in children
  • In children it tends to present as liver disease i.e. jaundice etc
  • In adults it tends to present as respiratory problems (dyspnoea)

Investigations:

  • A1AT concentrations- LOW
  • spirometry: obstructive picture

Management:

  • stop smoking
  • supportive: bronchodilators, physiotherapy
  • intravenous alpha1-antitrypsin protein concentrates
  • surgery: lung volume reduction surgery, lung transplantation
133
Q

What is liver failure?

What are the classifications?

Who does it more commonly affect?

List 5 clinical features

Describe the pathophysiology

What investigations would you request and expect to see?

What is the management?

List the complications

A

LIVER FAILURE

  • Hepatic failure occurs when the liver loses the ability to regenerate or repair, so that decompensation occurs. It is marked by:
    • Jaundice
    • Abnormal bleeding
    • Hepatic encephalopathy
    • Ascites
  • Histologically there is multiacinar necrosis involving a substantial part of the liver
  • Acute liver failure describes the rapid onset of hepatocellular dysfunction leading to a variety of systemic complications.

Classification:

  • hyperacute liver failure defined as development of encephalopathy within seven days of onset of illness
  • Fulminant hepatic failure (FHF) aka Acute liver failure occurs when the failure takes place within eight weeks of the onset of the underlying illness.
  • Late-onset hepatic failure (also called subacute FHF) occurs when there has been a gap of 8-26 weeks.
  • Chronic decompensated hepatic failure occurs when the latent period is over six months.

Aetiology:

  • paracetamol overdose – very common
  • alcohol
  • viral hepatitis (usually A or B)
  • CMV, EBV
  • acute fatty liver of pregnancy
  • Wilson’s disease or Alpha-1 antitrypsin deficiency
  • Hepatocellular carcinoma
  • Medications:
    • antibiotics (co-amoxiclav, ciprofloxacin, erythromycin), nitrofurantoin, isoniazid, statins and many more
  • Illicit drugs: Cocaine, ecstasy
  • Budd- Chiari syndrome
  • Reye’s syndrome

Features

  • jaundice
  • coagulopathy: raised prothrombin time
  • hypoalbuminaemia
  • hepatic encephalopathy (coma, altered behavoir/mood, confusion, liver flap)
  • Cerebral oedema with increased ICP, may produce papilloedema, hypertension, and bradycardia.
  • hepatomegaly, splenomelgaly
  • ascites
  • renal failure is common (‘hepatorenal syndrome’)

*remember that ‘liver function tests’ do not always accurately reflect the synthetic function of the liver. This is best assessed by looking at the prothrombin time and albumin level.

Differentials:

  • space-occupying lesions in the brain.
  • Cerebral infection - bacterial or viral.
  • Drug or alcohol intoxication.
  • Delirium tremens or Wernicke’s encephalopathy.
  • Metabolic upset such as hypoglycaemia, ketoacidosis, electrolyte imbalance, hypoxia, hypercapnia.

Investigations:

  • Blood:
  • FBC: likely to show iron-defiecy anaemia, thrombocytopenia
  • LFTs:
    • Hyperbilirubinemia
    • VERY High serum ALT & AST
  • Clotting screen
    • High INR
    • Low levels of coagulation factors and raised prothrombin time
  • Low glucose (since liver is glucose store, in form of glycogen)
  • Ammonia levels high
  • Blood cultures – to rule out infection
  • Imaging:
    • Electroencephalogram (EEG) useful in grading encephalopathy
    • Ultrasound or CT will define liver size and may indicate underlying liver pathology
    • Doppler ultrasound to see hepatic vein patency (Budd Chiari syndrome)
    • Avoid liver biopsy when there is compromised coagulation

Management:

  • Treat cause e.g. paracetamol poisoning give N-ACETYL-CYSTEINE - Monitor glucose levels and administer IV GLUCOSE if necessary
  • Signs of raised intracranial pressure give IV MANNITOL
  • Coagulopathy is managed with platelets concentrates, fresh frozen plasma, prothrombin complex concentrate ( IV BERIPLEX)
  • LACTULOSE, often with neomycin, is given to reduce ammonia production
  • Reduce haemorrhage risk by giving PPI e.g. LANSOPRAZOLE to reduce GI bleeds
  • Liver transplant

Complications:

  • Infection and sepsis - Spontaneous peritonitis is common, opportunistic infection and pneumonia may also occur.
  • Cerebral oedema may be associated with intracranial hypertension and death
  • Haemorrhage - Oesophageal varices need attention
  • AKI
  • Respiratory failure
134
Q

What is hepatic encephalopathy?

What are the classifications?

Who does it more commonly affect?

List 5 clinical features

Describe the pathophysiology

What investigations would you request and expect to see?

What is the management?

List the complications

A

HEPATIC ENCEPHALOPATHY

  • It may be seen in liver disease of any cause.
  • It is often associated with acute liver failure it may also be seen with chronic disease.

Aetiology:

  • As the liver fails, nitrogenous waste e.g. ammonia builds up in the circulation and passes to the brain which can result in permanent brain damage as ammonia is neurotoxic to the brain since it halts the Krebs cycle resulting in IRREPARABLE CELL DAMAGE and neural cell DEATH
  • Also as astrocytes try to clear ammonia (using a process involving the conversion of glutamate to glutamine), the excess glutamine causes an osmotic imbalance and a shift of fluid into these cells - hence cerebral oedema - resulting in damage

Precipitating factors:

  • infection e.g. spontaneous bacterial peritonitis
  • GI bleed
  • post transjugular intrahepatic portosystemic shunt
  • constipation
  • drugs: sedatives, diuretics, anti-depressants, antipsychotics
  • hypokalaemia
  • renal failure
  • increased dietary protein intake (uncommon)

Features

  • confusion, altered GCS
  • asterix: ‘liver flap’, arrhythmic negative myoclonus with a frequency of 3-5 Hz
  • hypothermia
  • fetor hepaticus
  • hyperventilation
  • constructional apraxia: inability to draw a 5-pointed star
  • triphasic slow waves on EEG
  • raised ammonia level (not commonly measured anymore)

Grading

  • Grade 0: subclinical; normal mental status, but minimal changes in memory, concentration, intellectual function, coordination.
  • Grade I: Irritability, mild confusion
  • Grade II: Confusion, inappropriate behaviour, drowsiness
  • Grade III: Incoherent, restless, marked confusion, speech is present but incomprehensible, disorientation to time and space
  • Grade IV: Coma with or without response to painful stimuli

Investigations:

  • Full septic screen
  • Ascitic tap to check for SBP
  • Digital rectal exam (DRE) to check for faecal impaction
  • Ammonia levels are raised and can help with diagnosis. The sample needs to be collected and then stored on ice and sent directly to the laboratory.
  • EEG
    • High-amplitude low-frequency waves and triphasic waves – not specific for hepatic encephalopathy.
  • MRI/CT can help to exclude other causes of altered mental function such as intracranial lesions
  • Visual evoked responses show classic patterns associated with hepatic encephalopathy.

Management:

  • treat any underlying precipitating cause
  • LACTULOSE - first-line
    • lactulose is thought to work by promoting the excretion of ammonia and increasing the metabolism of ammonia by gut bacteria
  • RIFAXIMIN for the secondary prophylaxis of hepatic encephalopathy
    • antibiotics such as rifaximin are thought to modulate the gut flora resulting in decreased ammonia production
  • other options include embolisation of portosystemic shunts and liver transplantation in selected patients
135
Q

What is oesophageal cancer?

What are the classifications?

Who does it more commonly affect?

List 5 clinical features

Describe the pathophysiology

What investigations would you request and expect to see?

What is the management?

List the complications

A

OESOPHAGEAL CANCER

  • Sixth most common cancer worldwide to cause death
  • Squamous cell carcinoma occurs in the middle third (40% of all oesophageal cancer) and in the upper third (15%) of the oesophagus
  • Adenocarcinomas occur in the lower third of the oesophagus (45%)
  • Adenocarcinomas are now more common than Squamous cell carcinoma
  • Carcinoma of the oesophagus occurs mainly in those aged 60-70 yrs
  • Incidence of SSC is decreasing in contrast to adenocarcinoma

Both SSC and adenocarcinoma are more common in MALES than females